LSAT and Law School Admissions Forum

Get expert LSAT preparation and law school admissions advice from PowerScore Test Preparation.

 Administrator
PowerScore Staff
  • PowerScore Staff
  • Posts: 8917
  • Joined: Feb 02, 2011
|
#41659
Complete Question Explanation
(The complete setup for this game can be found here: lsat/viewtopic.php?t=8572)

The correct answer choice is (E)

If S is the third program, we need to work with Templates 1B and 3A. In Template 1B, S is the third program after G and R; this template is fully determined, as shown below:
PT65_D11 LG Explanations_game_#4_#20_diagram 1.png
In Template 2A, S can only be the third program if it is shown at 2:00. Consequently, in accordance with the third condition, R must be shown at 1:00, leaving W to be shown at 2:30:
PT65_D11 LG Explanations_game_#4_#20_diagram 2.png
In both templates, W is the fourth program shown, validating answer choice (E).
You do not have the required permissions to view the files attached to this post.
 cfu1
  • Posts: 15
  • Joined: Mar 28, 2017
|
#34912
What is the best way to approach this problem? It took me a while to figure out that W has to be the fourth program.
 Adam Tyson
PowerScore Staff
  • PowerScore Staff
  • Posts: 5153
  • Joined: Apr 14, 2011
|
#35103
Thanks for the question, cfu1. In part, the answer depends on talking about the original setup of the game, so how you set it up will matter.

My base for this game is 6 time slots, beginning at 1pm with each slot representing a 30-minute slot - 1, 1:30, 2, 2:30, 3, 3:30.

Next, since this is a local question, draw a local diagram. If S is the third program, then it is either at 2:00, the third slot, or else it is at 2:30, the fourth slot (which would be true if G, the one-hour program that fills two of our slots, is one of the two programs before it). Draw these two possibilities out, something like this:

_ _ S _ _ _

_ _ _ S _ _

In both case, R must be before S per the 3rd rule. In the top scenario, what else could be in the first three? Not G, because there is no room. Not W, because then W would be before T and would have to be immediately before T per the 4th rule. That leaves only T, which must start on the half hour at 1:30 per the 2nd rule. So far, then, we have this:

RTS _ _ _

To finish that up, G has to start on the hour, and the only place that can happen is at 3:00, taking up the final hour. That leaves W to fill in the 2:30 spot, like this:

RTSW[GG}

In the second possible scenario, if G is one of the first two programs, it must start at 1:00, and you begin like this:

[GG}RS _ _

There are only two slots left for T and W to fill, and T has to start on the half hour at 3:30, forcing W to the 3:00 slot:

[GG}RSWT

Now with these two possible solutions when S is the third program, we can see that one other thing is constant, and that is that W is the 4th program, either at 2:30 or else at 3:00. Answers A through D are all COULD be true answers, but only answer E MUST be true.

The local nature of this question, setting up an "if" scenario that adds a restriction not in the original rules, usually calls for a local diagram. When in doubt, draw it out! The time spent on the diagram will usually pay off by giving you the confidence to pick the right answer very quickly once you get to reviewing the answers.

Get the most out of your LSAT Prep Plus subscription.

Analyze and track your performance with our Testing and Analytics Package.